Proof involving polynomial roots

The name of the pictureThe name of the pictureThe name of the pictureClash Royale CLAN TAG#URR8PPP











up vote
7
down vote

favorite
2












From USAMO 1977:
"If $a$ and $b$ are two roots of $x^4+x^3-1=0$, prove that $ab$ is a root of $x^6+x^4+x^3-x^2-1=0$."



The solutions I've seen for this problem all involve manipulating Vieta's equations relating polynomial roots and coefficients. In particular, the solutions feel a bit like they've been plucked out of thin air, in that I don't understand their motivation. So I'd be interested to see some alternate solutions/extra explanation.



I instead tried to construct $(ab)^6+(ab)^4+(ab)^3-(ab)^2-1=0$ using $a^4+a^3-1=0$ and $b^4+b^3-1=0$, but was unsuccessful.







share|cite|improve this question















  • 1




    You can see here artofproblemsolving.com/wiki/…
    – Cesareo
    Jul 20 at 7:42










  • By using your approach, how do you use the fact that $anot=b$?
    – Robert Z
    Jul 20 at 7:43










  • thanks, but I have already looked at that solution, and didn't really understand the motivation
    – Merk Zockerborg
    Jul 20 at 7:44










  • @RobertZ I subtracted the two equations from each other and factored out the $a-b$
    – Merk Zockerborg
    Jul 20 at 7:44














up vote
7
down vote

favorite
2












From USAMO 1977:
"If $a$ and $b$ are two roots of $x^4+x^3-1=0$, prove that $ab$ is a root of $x^6+x^4+x^3-x^2-1=0$."



The solutions I've seen for this problem all involve manipulating Vieta's equations relating polynomial roots and coefficients. In particular, the solutions feel a bit like they've been plucked out of thin air, in that I don't understand their motivation. So I'd be interested to see some alternate solutions/extra explanation.



I instead tried to construct $(ab)^6+(ab)^4+(ab)^3-(ab)^2-1=0$ using $a^4+a^3-1=0$ and $b^4+b^3-1=0$, but was unsuccessful.







share|cite|improve this question















  • 1




    You can see here artofproblemsolving.com/wiki/…
    – Cesareo
    Jul 20 at 7:42










  • By using your approach, how do you use the fact that $anot=b$?
    – Robert Z
    Jul 20 at 7:43










  • thanks, but I have already looked at that solution, and didn't really understand the motivation
    – Merk Zockerborg
    Jul 20 at 7:44










  • @RobertZ I subtracted the two equations from each other and factored out the $a-b$
    – Merk Zockerborg
    Jul 20 at 7:44












up vote
7
down vote

favorite
2









up vote
7
down vote

favorite
2






2





From USAMO 1977:
"If $a$ and $b$ are two roots of $x^4+x^3-1=0$, prove that $ab$ is a root of $x^6+x^4+x^3-x^2-1=0$."



The solutions I've seen for this problem all involve manipulating Vieta's equations relating polynomial roots and coefficients. In particular, the solutions feel a bit like they've been plucked out of thin air, in that I don't understand their motivation. So I'd be interested to see some alternate solutions/extra explanation.



I instead tried to construct $(ab)^6+(ab)^4+(ab)^3-(ab)^2-1=0$ using $a^4+a^3-1=0$ and $b^4+b^3-1=0$, but was unsuccessful.







share|cite|improve this question











From USAMO 1977:
"If $a$ and $b$ are two roots of $x^4+x^3-1=0$, prove that $ab$ is a root of $x^6+x^4+x^3-x^2-1=0$."



The solutions I've seen for this problem all involve manipulating Vieta's equations relating polynomial roots and coefficients. In particular, the solutions feel a bit like they've been plucked out of thin air, in that I don't understand their motivation. So I'd be interested to see some alternate solutions/extra explanation.



I instead tried to construct $(ab)^6+(ab)^4+(ab)^3-(ab)^2-1=0$ using $a^4+a^3-1=0$ and $b^4+b^3-1=0$, but was unsuccessful.









share|cite|improve this question










share|cite|improve this question




share|cite|improve this question









asked Jul 20 at 7:28









Merk Zockerborg

1429




1429







  • 1




    You can see here artofproblemsolving.com/wiki/…
    – Cesareo
    Jul 20 at 7:42










  • By using your approach, how do you use the fact that $anot=b$?
    – Robert Z
    Jul 20 at 7:43










  • thanks, but I have already looked at that solution, and didn't really understand the motivation
    – Merk Zockerborg
    Jul 20 at 7:44










  • @RobertZ I subtracted the two equations from each other and factored out the $a-b$
    – Merk Zockerborg
    Jul 20 at 7:44












  • 1




    You can see here artofproblemsolving.com/wiki/…
    – Cesareo
    Jul 20 at 7:42










  • By using your approach, how do you use the fact that $anot=b$?
    – Robert Z
    Jul 20 at 7:43










  • thanks, but I have already looked at that solution, and didn't really understand the motivation
    – Merk Zockerborg
    Jul 20 at 7:44










  • @RobertZ I subtracted the two equations from each other and factored out the $a-b$
    – Merk Zockerborg
    Jul 20 at 7:44







1




1




You can see here artofproblemsolving.com/wiki/…
– Cesareo
Jul 20 at 7:42




You can see here artofproblemsolving.com/wiki/…
– Cesareo
Jul 20 at 7:42












By using your approach, how do you use the fact that $anot=b$?
– Robert Z
Jul 20 at 7:43




By using your approach, how do you use the fact that $anot=b$?
– Robert Z
Jul 20 at 7:43












thanks, but I have already looked at that solution, and didn't really understand the motivation
– Merk Zockerborg
Jul 20 at 7:44




thanks, but I have already looked at that solution, and didn't really understand the motivation
– Merk Zockerborg
Jul 20 at 7:44












@RobertZ I subtracted the two equations from each other and factored out the $a-b$
– Merk Zockerborg
Jul 20 at 7:44




@RobertZ I subtracted the two equations from each other and factored out the $a-b$
– Merk Zockerborg
Jul 20 at 7:44










3 Answers
3






active

oldest

votes

















up vote
5
down vote



accepted










Considering



beginalign
x^4+x^3-1 &= (x-a)(x-b)(x-c)(x-d) \
S_1 &= a+b+c+d \
&= -1 \
S_2 &= ab+ac+ad+bc+bd+cd \
&= 0 \
S_3 &= bcd+acd+abd+abc \
&= 0 \
S_4 &= abcd \
&= -1 endalign




The sextic has a nice symmetry so that all the roots are $ab$, $ac$, $ad$, $bc$, $bd$ and $cd$.




Now, using SymmetricReduction[(x-a b)(x-a c)(x-a d)(x-b c)(x-b d)(x-c d),a,b,c,d] in Wolfram Alpha or so, we have:



beginalign
f(x) &= (x-colorredab)(x-colorredac)(x-colorredad)
(x-colorredbc)(x-colorredbd)(x-colorredcd) \
&= x^6-S_2 x^5+(S_1 S_3-S_4)x^4+(2S_2 S_4-S_3^2-S_1^2 S_4)x^3 \
& quad +(S_1 S_3 S_4-S_4^2)x^2-S_2 S_4^2 x+S_4^3 \
&= x^6+x^4+x^3-x^2-1
endalign






share|cite|improve this answer























  • +1 Beautiful insight.
    – Cave Johnson
    Jul 21 at 1:42










  • Can you explain why "The sextic has a nice symmetry so that all the roots are ab, ac, ad, bc, bd and cd" is true?
    – Merk Zockerborg
    Jul 21 at 16:36











  • Since $a$ and $b$ are not specified for which are which, all $a$, $b$, $c$ and $d$ have equal footings. That's to say any two of them will do. Hence there're $binom42=6$ combinations.
    – Ng Chung Tak
    Jul 21 at 19:24











  • I see, so you're working backwards, thanks
    – Merk Zockerborg
    Jul 22 at 14:59

















up vote
3
down vote














I instead tried to construct $(ab)^6+(ab)^4+(ab)^3-(ab)^2-1=0$ using $,ldots$




Following up on OP's approach, let $,ab=p,$ and, for symmetry, $,a+b=s,$. To begin with:



$$
begincases
beginalign
a^4 + a^3 - 1 &= 0 \
b^4 + b^3 - 1 &= 0
endalign
endcases tag1
$$



Subtracting the two equations in $,(1),$, and using that $,a ne b,$:



$$requirecancel
beginalign
a^4-b^4+a^3-b^3 = 0 ;;&implies;; cancel(a-b)(a+b)(a^2+b^2) + cancel(a-b)(a^2+ab+b^2) = 0 \
&implies;; s(s^2-2p) + s^2 - p = 0 \
&implies;; s^3 + s^2 - (2s+1)p = 0 tag2
endalign
$$



Adding the two equations in $,(1),$:



$$
beginalign
a^4+b^4+a^3+b^3 - 2 = 0 ;;&implies;; big((a+b)^4 - 4ab(a^2+b^2)-6a^2b^2big) \
&quadquadquadquad+big((a+b)^3-3ab(a+b)big)-2=0 \
&implies;; s^4 - 4p(s^2-2p)-6p^2+s^3-3ps-2 = 0 \
&implies;; s^4 + s^3 -4ps^2-3ps+2p^2-2=0 tag3
endalign
$$



At this point, the equation satisfied by $,p,$ can be derived by eliminating $,s,$ between $,(2),$ and $,(3),$. This is usually better left to a CAS since the calculations can be tedious, and e.g. WA gives resultant[$s^3 + s^2 - (2s+1)p$, $s^4 + s^3 - 4ps^2 - 3ps + 2p^2 - 2, s$] = $8 (p^6 + p^4 + p^3 - p^2 - 1)$.



It can still be done by hand, though. Substituting $,s^4+s^3=s cdot (2s+1)p,$ from $,(2),$ into $,(3),$:



$$
beginalign
s(2s+1)p -4ps^2-3ps+2p^2-2=0 ;;&implies;; -2ps^2 - 2ps+ 2p^2 - 2 = 0 \
&implies;; ps^2+ps-p^2+1 = 0 tag4
endalign
$$



Eliminating once again $,s(s+1),$ between $,(2),$ and $,(4),$ by $,s cdot (4) - p cdot (2),$ gives:



$$
beginalign
0 &= s cdot big(cancelps(s+1)-p^2+1big) - p cdot big(cancels^2(s+1) - (2s+1)pbig) = (p^2+1)s+p^2 tag5
endalign
$$



Then, substituting $,s = frac-p^2p^2+1,$ back in $,(4),$ gives in the end the sextic $,p^6 + p^4 + p^3 - p^2 - 1 = 0,$.






share|cite|improve this answer




























    up vote
    0
    down vote













    I think that the problem is wrong as written, because if $a=b$, then $ab=a^2$ has minimal polynomial $x^4 - x^3 - 2 x^2 + 1$, which doesn't divide $x^6+x^4+x^3-x^2-1$ (the reminder is $7 x^3 + 6 x^2 - x - 5$).



    I guess that it was meant that $aneq b$. Then, one may apply Galois theory to get a more straightforward way to compute the minimal polynomial of $ab$ and then to check whether or not it divides $x^6+x^4+x^3-x^2-1$ (it is a root iff the minimal polynomial divides...)



    Here is a sketch how to do so: You prove that $mathbbQ(a,b)$ has degree $12$ and a basis $a^ib^j$ with $i=0,1,2,3$ and $j=0,1,2$. Then you write a matrix AB for multiplication by $ab$ in this basis (usually it is easier to write a matrix $A$ for $a$, a matrix $B$ for $b$, and to multiply them). The minimal polynomial polynomial will be that of the matrix $AB$.



    The downside is that one needs to work with $12times 12$ matrices. But it is not too bad, since those matrices are essentially block-matrices.






    share|cite|improve this answer





















      Your Answer




      StackExchange.ifUsing("editor", function ()
      return StackExchange.using("mathjaxEditing", function ()
      StackExchange.MarkdownEditor.creationCallbacks.add(function (editor, postfix)
      StackExchange.mathjaxEditing.prepareWmdForMathJax(editor, postfix, [["$", "$"], ["\\(","\\)"]]);
      );
      );
      , "mathjax-editing");

      StackExchange.ready(function()
      var channelOptions =
      tags: "".split(" "),
      id: "69"
      ;
      initTagRenderer("".split(" "), "".split(" "), channelOptions);

      StackExchange.using("externalEditor", function()
      // Have to fire editor after snippets, if snippets enabled
      if (StackExchange.settings.snippets.snippetsEnabled)
      StackExchange.using("snippets", function()
      createEditor();
      );

      else
      createEditor();

      );

      function createEditor()
      StackExchange.prepareEditor(
      heartbeatType: 'answer',
      convertImagesToLinks: true,
      noModals: false,
      showLowRepImageUploadWarning: true,
      reputationToPostImages: 10,
      bindNavPrevention: true,
      postfix: "",
      noCode: true, onDemand: true,
      discardSelector: ".discard-answer"
      ,immediatelyShowMarkdownHelp:true
      );



      );








       

      draft saved


      draft discarded


















      StackExchange.ready(
      function ()
      StackExchange.openid.initPostLogin('.new-post-login', 'https%3a%2f%2fmath.stackexchange.com%2fquestions%2f2857380%2fproof-involving-polynomial-roots%23new-answer', 'question_page');

      );

      Post as a guest






























      3 Answers
      3






      active

      oldest

      votes








      3 Answers
      3






      active

      oldest

      votes









      active

      oldest

      votes






      active

      oldest

      votes








      up vote
      5
      down vote



      accepted










      Considering



      beginalign
      x^4+x^3-1 &= (x-a)(x-b)(x-c)(x-d) \
      S_1 &= a+b+c+d \
      &= -1 \
      S_2 &= ab+ac+ad+bc+bd+cd \
      &= 0 \
      S_3 &= bcd+acd+abd+abc \
      &= 0 \
      S_4 &= abcd \
      &= -1 endalign




      The sextic has a nice symmetry so that all the roots are $ab$, $ac$, $ad$, $bc$, $bd$ and $cd$.




      Now, using SymmetricReduction[(x-a b)(x-a c)(x-a d)(x-b c)(x-b d)(x-c d),a,b,c,d] in Wolfram Alpha or so, we have:



      beginalign
      f(x) &= (x-colorredab)(x-colorredac)(x-colorredad)
      (x-colorredbc)(x-colorredbd)(x-colorredcd) \
      &= x^6-S_2 x^5+(S_1 S_3-S_4)x^4+(2S_2 S_4-S_3^2-S_1^2 S_4)x^3 \
      & quad +(S_1 S_3 S_4-S_4^2)x^2-S_2 S_4^2 x+S_4^3 \
      &= x^6+x^4+x^3-x^2-1
      endalign






      share|cite|improve this answer























      • +1 Beautiful insight.
        – Cave Johnson
        Jul 21 at 1:42










      • Can you explain why "The sextic has a nice symmetry so that all the roots are ab, ac, ad, bc, bd and cd" is true?
        – Merk Zockerborg
        Jul 21 at 16:36











      • Since $a$ and $b$ are not specified for which are which, all $a$, $b$, $c$ and $d$ have equal footings. That's to say any two of them will do. Hence there're $binom42=6$ combinations.
        – Ng Chung Tak
        Jul 21 at 19:24











      • I see, so you're working backwards, thanks
        – Merk Zockerborg
        Jul 22 at 14:59














      up vote
      5
      down vote



      accepted










      Considering



      beginalign
      x^4+x^3-1 &= (x-a)(x-b)(x-c)(x-d) \
      S_1 &= a+b+c+d \
      &= -1 \
      S_2 &= ab+ac+ad+bc+bd+cd \
      &= 0 \
      S_3 &= bcd+acd+abd+abc \
      &= 0 \
      S_4 &= abcd \
      &= -1 endalign




      The sextic has a nice symmetry so that all the roots are $ab$, $ac$, $ad$, $bc$, $bd$ and $cd$.




      Now, using SymmetricReduction[(x-a b)(x-a c)(x-a d)(x-b c)(x-b d)(x-c d),a,b,c,d] in Wolfram Alpha or so, we have:



      beginalign
      f(x) &= (x-colorredab)(x-colorredac)(x-colorredad)
      (x-colorredbc)(x-colorredbd)(x-colorredcd) \
      &= x^6-S_2 x^5+(S_1 S_3-S_4)x^4+(2S_2 S_4-S_3^2-S_1^2 S_4)x^3 \
      & quad +(S_1 S_3 S_4-S_4^2)x^2-S_2 S_4^2 x+S_4^3 \
      &= x^6+x^4+x^3-x^2-1
      endalign






      share|cite|improve this answer























      • +1 Beautiful insight.
        – Cave Johnson
        Jul 21 at 1:42










      • Can you explain why "The sextic has a nice symmetry so that all the roots are ab, ac, ad, bc, bd and cd" is true?
        – Merk Zockerborg
        Jul 21 at 16:36











      • Since $a$ and $b$ are not specified for which are which, all $a$, $b$, $c$ and $d$ have equal footings. That's to say any two of them will do. Hence there're $binom42=6$ combinations.
        – Ng Chung Tak
        Jul 21 at 19:24











      • I see, so you're working backwards, thanks
        – Merk Zockerborg
        Jul 22 at 14:59












      up vote
      5
      down vote



      accepted







      up vote
      5
      down vote



      accepted






      Considering



      beginalign
      x^4+x^3-1 &= (x-a)(x-b)(x-c)(x-d) \
      S_1 &= a+b+c+d \
      &= -1 \
      S_2 &= ab+ac+ad+bc+bd+cd \
      &= 0 \
      S_3 &= bcd+acd+abd+abc \
      &= 0 \
      S_4 &= abcd \
      &= -1 endalign




      The sextic has a nice symmetry so that all the roots are $ab$, $ac$, $ad$, $bc$, $bd$ and $cd$.




      Now, using SymmetricReduction[(x-a b)(x-a c)(x-a d)(x-b c)(x-b d)(x-c d),a,b,c,d] in Wolfram Alpha or so, we have:



      beginalign
      f(x) &= (x-colorredab)(x-colorredac)(x-colorredad)
      (x-colorredbc)(x-colorredbd)(x-colorredcd) \
      &= x^6-S_2 x^5+(S_1 S_3-S_4)x^4+(2S_2 S_4-S_3^2-S_1^2 S_4)x^3 \
      & quad +(S_1 S_3 S_4-S_4^2)x^2-S_2 S_4^2 x+S_4^3 \
      &= x^6+x^4+x^3-x^2-1
      endalign






      share|cite|improve this answer















      Considering



      beginalign
      x^4+x^3-1 &= (x-a)(x-b)(x-c)(x-d) \
      S_1 &= a+b+c+d \
      &= -1 \
      S_2 &= ab+ac+ad+bc+bd+cd \
      &= 0 \
      S_3 &= bcd+acd+abd+abc \
      &= 0 \
      S_4 &= abcd \
      &= -1 endalign




      The sextic has a nice symmetry so that all the roots are $ab$, $ac$, $ad$, $bc$, $bd$ and $cd$.




      Now, using SymmetricReduction[(x-a b)(x-a c)(x-a d)(x-b c)(x-b d)(x-c d),a,b,c,d] in Wolfram Alpha or so, we have:



      beginalign
      f(x) &= (x-colorredab)(x-colorredac)(x-colorredad)
      (x-colorredbc)(x-colorredbd)(x-colorredcd) \
      &= x^6-S_2 x^5+(S_1 S_3-S_4)x^4+(2S_2 S_4-S_3^2-S_1^2 S_4)x^3 \
      & quad +(S_1 S_3 S_4-S_4^2)x^2-S_2 S_4^2 x+S_4^3 \
      &= x^6+x^4+x^3-x^2-1
      endalign







      share|cite|improve this answer















      share|cite|improve this answer



      share|cite|improve this answer








      edited Jul 20 at 16:48


























      answered Jul 20 at 15:05









      Ng Chung Tak

      13k31130




      13k31130











      • +1 Beautiful insight.
        – Cave Johnson
        Jul 21 at 1:42










      • Can you explain why "The sextic has a nice symmetry so that all the roots are ab, ac, ad, bc, bd and cd" is true?
        – Merk Zockerborg
        Jul 21 at 16:36











      • Since $a$ and $b$ are not specified for which are which, all $a$, $b$, $c$ and $d$ have equal footings. That's to say any two of them will do. Hence there're $binom42=6$ combinations.
        – Ng Chung Tak
        Jul 21 at 19:24











      • I see, so you're working backwards, thanks
        – Merk Zockerborg
        Jul 22 at 14:59
















      • +1 Beautiful insight.
        – Cave Johnson
        Jul 21 at 1:42










      • Can you explain why "The sextic has a nice symmetry so that all the roots are ab, ac, ad, bc, bd and cd" is true?
        – Merk Zockerborg
        Jul 21 at 16:36











      • Since $a$ and $b$ are not specified for which are which, all $a$, $b$, $c$ and $d$ have equal footings. That's to say any two of them will do. Hence there're $binom42=6$ combinations.
        – Ng Chung Tak
        Jul 21 at 19:24











      • I see, so you're working backwards, thanks
        – Merk Zockerborg
        Jul 22 at 14:59















      +1 Beautiful insight.
      – Cave Johnson
      Jul 21 at 1:42




      +1 Beautiful insight.
      – Cave Johnson
      Jul 21 at 1:42












      Can you explain why "The sextic has a nice symmetry so that all the roots are ab, ac, ad, bc, bd and cd" is true?
      – Merk Zockerborg
      Jul 21 at 16:36





      Can you explain why "The sextic has a nice symmetry so that all the roots are ab, ac, ad, bc, bd and cd" is true?
      – Merk Zockerborg
      Jul 21 at 16:36













      Since $a$ and $b$ are not specified for which are which, all $a$, $b$, $c$ and $d$ have equal footings. That's to say any two of them will do. Hence there're $binom42=6$ combinations.
      – Ng Chung Tak
      Jul 21 at 19:24





      Since $a$ and $b$ are not specified for which are which, all $a$, $b$, $c$ and $d$ have equal footings. That's to say any two of them will do. Hence there're $binom42=6$ combinations.
      – Ng Chung Tak
      Jul 21 at 19:24













      I see, so you're working backwards, thanks
      – Merk Zockerborg
      Jul 22 at 14:59




      I see, so you're working backwards, thanks
      – Merk Zockerborg
      Jul 22 at 14:59










      up vote
      3
      down vote














      I instead tried to construct $(ab)^6+(ab)^4+(ab)^3-(ab)^2-1=0$ using $,ldots$




      Following up on OP's approach, let $,ab=p,$ and, for symmetry, $,a+b=s,$. To begin with:



      $$
      begincases
      beginalign
      a^4 + a^3 - 1 &= 0 \
      b^4 + b^3 - 1 &= 0
      endalign
      endcases tag1
      $$



      Subtracting the two equations in $,(1),$, and using that $,a ne b,$:



      $$requirecancel
      beginalign
      a^4-b^4+a^3-b^3 = 0 ;;&implies;; cancel(a-b)(a+b)(a^2+b^2) + cancel(a-b)(a^2+ab+b^2) = 0 \
      &implies;; s(s^2-2p) + s^2 - p = 0 \
      &implies;; s^3 + s^2 - (2s+1)p = 0 tag2
      endalign
      $$



      Adding the two equations in $,(1),$:



      $$
      beginalign
      a^4+b^4+a^3+b^3 - 2 = 0 ;;&implies;; big((a+b)^4 - 4ab(a^2+b^2)-6a^2b^2big) \
      &quadquadquadquad+big((a+b)^3-3ab(a+b)big)-2=0 \
      &implies;; s^4 - 4p(s^2-2p)-6p^2+s^3-3ps-2 = 0 \
      &implies;; s^4 + s^3 -4ps^2-3ps+2p^2-2=0 tag3
      endalign
      $$



      At this point, the equation satisfied by $,p,$ can be derived by eliminating $,s,$ between $,(2),$ and $,(3),$. This is usually better left to a CAS since the calculations can be tedious, and e.g. WA gives resultant[$s^3 + s^2 - (2s+1)p$, $s^4 + s^3 - 4ps^2 - 3ps + 2p^2 - 2, s$] = $8 (p^6 + p^4 + p^3 - p^2 - 1)$.



      It can still be done by hand, though. Substituting $,s^4+s^3=s cdot (2s+1)p,$ from $,(2),$ into $,(3),$:



      $$
      beginalign
      s(2s+1)p -4ps^2-3ps+2p^2-2=0 ;;&implies;; -2ps^2 - 2ps+ 2p^2 - 2 = 0 \
      &implies;; ps^2+ps-p^2+1 = 0 tag4
      endalign
      $$



      Eliminating once again $,s(s+1),$ between $,(2),$ and $,(4),$ by $,s cdot (4) - p cdot (2),$ gives:



      $$
      beginalign
      0 &= s cdot big(cancelps(s+1)-p^2+1big) - p cdot big(cancels^2(s+1) - (2s+1)pbig) = (p^2+1)s+p^2 tag5
      endalign
      $$



      Then, substituting $,s = frac-p^2p^2+1,$ back in $,(4),$ gives in the end the sextic $,p^6 + p^4 + p^3 - p^2 - 1 = 0,$.






      share|cite|improve this answer

























        up vote
        3
        down vote














        I instead tried to construct $(ab)^6+(ab)^4+(ab)^3-(ab)^2-1=0$ using $,ldots$




        Following up on OP's approach, let $,ab=p,$ and, for symmetry, $,a+b=s,$. To begin with:



        $$
        begincases
        beginalign
        a^4 + a^3 - 1 &= 0 \
        b^4 + b^3 - 1 &= 0
        endalign
        endcases tag1
        $$



        Subtracting the two equations in $,(1),$, and using that $,a ne b,$:



        $$requirecancel
        beginalign
        a^4-b^4+a^3-b^3 = 0 ;;&implies;; cancel(a-b)(a+b)(a^2+b^2) + cancel(a-b)(a^2+ab+b^2) = 0 \
        &implies;; s(s^2-2p) + s^2 - p = 0 \
        &implies;; s^3 + s^2 - (2s+1)p = 0 tag2
        endalign
        $$



        Adding the two equations in $,(1),$:



        $$
        beginalign
        a^4+b^4+a^3+b^3 - 2 = 0 ;;&implies;; big((a+b)^4 - 4ab(a^2+b^2)-6a^2b^2big) \
        &quadquadquadquad+big((a+b)^3-3ab(a+b)big)-2=0 \
        &implies;; s^4 - 4p(s^2-2p)-6p^2+s^3-3ps-2 = 0 \
        &implies;; s^4 + s^3 -4ps^2-3ps+2p^2-2=0 tag3
        endalign
        $$



        At this point, the equation satisfied by $,p,$ can be derived by eliminating $,s,$ between $,(2),$ and $,(3),$. This is usually better left to a CAS since the calculations can be tedious, and e.g. WA gives resultant[$s^3 + s^2 - (2s+1)p$, $s^4 + s^3 - 4ps^2 - 3ps + 2p^2 - 2, s$] = $8 (p^6 + p^4 + p^3 - p^2 - 1)$.



        It can still be done by hand, though. Substituting $,s^4+s^3=s cdot (2s+1)p,$ from $,(2),$ into $,(3),$:



        $$
        beginalign
        s(2s+1)p -4ps^2-3ps+2p^2-2=0 ;;&implies;; -2ps^2 - 2ps+ 2p^2 - 2 = 0 \
        &implies;; ps^2+ps-p^2+1 = 0 tag4
        endalign
        $$



        Eliminating once again $,s(s+1),$ between $,(2),$ and $,(4),$ by $,s cdot (4) - p cdot (2),$ gives:



        $$
        beginalign
        0 &= s cdot big(cancelps(s+1)-p^2+1big) - p cdot big(cancels^2(s+1) - (2s+1)pbig) = (p^2+1)s+p^2 tag5
        endalign
        $$



        Then, substituting $,s = frac-p^2p^2+1,$ back in $,(4),$ gives in the end the sextic $,p^6 + p^4 + p^3 - p^2 - 1 = 0,$.






        share|cite|improve this answer























          up vote
          3
          down vote










          up vote
          3
          down vote










          I instead tried to construct $(ab)^6+(ab)^4+(ab)^3-(ab)^2-1=0$ using $,ldots$




          Following up on OP's approach, let $,ab=p,$ and, for symmetry, $,a+b=s,$. To begin with:



          $$
          begincases
          beginalign
          a^4 + a^3 - 1 &= 0 \
          b^4 + b^3 - 1 &= 0
          endalign
          endcases tag1
          $$



          Subtracting the two equations in $,(1),$, and using that $,a ne b,$:



          $$requirecancel
          beginalign
          a^4-b^4+a^3-b^3 = 0 ;;&implies;; cancel(a-b)(a+b)(a^2+b^2) + cancel(a-b)(a^2+ab+b^2) = 0 \
          &implies;; s(s^2-2p) + s^2 - p = 0 \
          &implies;; s^3 + s^2 - (2s+1)p = 0 tag2
          endalign
          $$



          Adding the two equations in $,(1),$:



          $$
          beginalign
          a^4+b^4+a^3+b^3 - 2 = 0 ;;&implies;; big((a+b)^4 - 4ab(a^2+b^2)-6a^2b^2big) \
          &quadquadquadquad+big((a+b)^3-3ab(a+b)big)-2=0 \
          &implies;; s^4 - 4p(s^2-2p)-6p^2+s^3-3ps-2 = 0 \
          &implies;; s^4 + s^3 -4ps^2-3ps+2p^2-2=0 tag3
          endalign
          $$



          At this point, the equation satisfied by $,p,$ can be derived by eliminating $,s,$ between $,(2),$ and $,(3),$. This is usually better left to a CAS since the calculations can be tedious, and e.g. WA gives resultant[$s^3 + s^2 - (2s+1)p$, $s^4 + s^3 - 4ps^2 - 3ps + 2p^2 - 2, s$] = $8 (p^6 + p^4 + p^3 - p^2 - 1)$.



          It can still be done by hand, though. Substituting $,s^4+s^3=s cdot (2s+1)p,$ from $,(2),$ into $,(3),$:



          $$
          beginalign
          s(2s+1)p -4ps^2-3ps+2p^2-2=0 ;;&implies;; -2ps^2 - 2ps+ 2p^2 - 2 = 0 \
          &implies;; ps^2+ps-p^2+1 = 0 tag4
          endalign
          $$



          Eliminating once again $,s(s+1),$ between $,(2),$ and $,(4),$ by $,s cdot (4) - p cdot (2),$ gives:



          $$
          beginalign
          0 &= s cdot big(cancelps(s+1)-p^2+1big) - p cdot big(cancels^2(s+1) - (2s+1)pbig) = (p^2+1)s+p^2 tag5
          endalign
          $$



          Then, substituting $,s = frac-p^2p^2+1,$ back in $,(4),$ gives in the end the sextic $,p^6 + p^4 + p^3 - p^2 - 1 = 0,$.






          share|cite|improve this answer














          I instead tried to construct $(ab)^6+(ab)^4+(ab)^3-(ab)^2-1=0$ using $,ldots$




          Following up on OP's approach, let $,ab=p,$ and, for symmetry, $,a+b=s,$. To begin with:



          $$
          begincases
          beginalign
          a^4 + a^3 - 1 &= 0 \
          b^4 + b^3 - 1 &= 0
          endalign
          endcases tag1
          $$



          Subtracting the two equations in $,(1),$, and using that $,a ne b,$:



          $$requirecancel
          beginalign
          a^4-b^4+a^3-b^3 = 0 ;;&implies;; cancel(a-b)(a+b)(a^2+b^2) + cancel(a-b)(a^2+ab+b^2) = 0 \
          &implies;; s(s^2-2p) + s^2 - p = 0 \
          &implies;; s^3 + s^2 - (2s+1)p = 0 tag2
          endalign
          $$



          Adding the two equations in $,(1),$:



          $$
          beginalign
          a^4+b^4+a^3+b^3 - 2 = 0 ;;&implies;; big((a+b)^4 - 4ab(a^2+b^2)-6a^2b^2big) \
          &quadquadquadquad+big((a+b)^3-3ab(a+b)big)-2=0 \
          &implies;; s^4 - 4p(s^2-2p)-6p^2+s^3-3ps-2 = 0 \
          &implies;; s^4 + s^3 -4ps^2-3ps+2p^2-2=0 tag3
          endalign
          $$



          At this point, the equation satisfied by $,p,$ can be derived by eliminating $,s,$ between $,(2),$ and $,(3),$. This is usually better left to a CAS since the calculations can be tedious, and e.g. WA gives resultant[$s^3 + s^2 - (2s+1)p$, $s^4 + s^3 - 4ps^2 - 3ps + 2p^2 - 2, s$] = $8 (p^6 + p^4 + p^3 - p^2 - 1)$.



          It can still be done by hand, though. Substituting $,s^4+s^3=s cdot (2s+1)p,$ from $,(2),$ into $,(3),$:



          $$
          beginalign
          s(2s+1)p -4ps^2-3ps+2p^2-2=0 ;;&implies;; -2ps^2 - 2ps+ 2p^2 - 2 = 0 \
          &implies;; ps^2+ps-p^2+1 = 0 tag4
          endalign
          $$



          Eliminating once again $,s(s+1),$ between $,(2),$ and $,(4),$ by $,s cdot (4) - p cdot (2),$ gives:



          $$
          beginalign
          0 &= s cdot big(cancelps(s+1)-p^2+1big) - p cdot big(cancels^2(s+1) - (2s+1)pbig) = (p^2+1)s+p^2 tag5
          endalign
          $$



          Then, substituting $,s = frac-p^2p^2+1,$ back in $,(4),$ gives in the end the sextic $,p^6 + p^4 + p^3 - p^2 - 1 = 0,$.







          share|cite|improve this answer













          share|cite|improve this answer



          share|cite|improve this answer











          answered Jul 21 at 1:36









          dxiv

          54.2k64797




          54.2k64797




















              up vote
              0
              down vote













              I think that the problem is wrong as written, because if $a=b$, then $ab=a^2$ has minimal polynomial $x^4 - x^3 - 2 x^2 + 1$, which doesn't divide $x^6+x^4+x^3-x^2-1$ (the reminder is $7 x^3 + 6 x^2 - x - 5$).



              I guess that it was meant that $aneq b$. Then, one may apply Galois theory to get a more straightforward way to compute the minimal polynomial of $ab$ and then to check whether or not it divides $x^6+x^4+x^3-x^2-1$ (it is a root iff the minimal polynomial divides...)



              Here is a sketch how to do so: You prove that $mathbbQ(a,b)$ has degree $12$ and a basis $a^ib^j$ with $i=0,1,2,3$ and $j=0,1,2$. Then you write a matrix AB for multiplication by $ab$ in this basis (usually it is easier to write a matrix $A$ for $a$, a matrix $B$ for $b$, and to multiply them). The minimal polynomial polynomial will be that of the matrix $AB$.



              The downside is that one needs to work with $12times 12$ matrices. But it is not too bad, since those matrices are essentially block-matrices.






              share|cite|improve this answer

























                up vote
                0
                down vote













                I think that the problem is wrong as written, because if $a=b$, then $ab=a^2$ has minimal polynomial $x^4 - x^3 - 2 x^2 + 1$, which doesn't divide $x^6+x^4+x^3-x^2-1$ (the reminder is $7 x^3 + 6 x^2 - x - 5$).



                I guess that it was meant that $aneq b$. Then, one may apply Galois theory to get a more straightforward way to compute the minimal polynomial of $ab$ and then to check whether or not it divides $x^6+x^4+x^3-x^2-1$ (it is a root iff the minimal polynomial divides...)



                Here is a sketch how to do so: You prove that $mathbbQ(a,b)$ has degree $12$ and a basis $a^ib^j$ with $i=0,1,2,3$ and $j=0,1,2$. Then you write a matrix AB for multiplication by $ab$ in this basis (usually it is easier to write a matrix $A$ for $a$, a matrix $B$ for $b$, and to multiply them). The minimal polynomial polynomial will be that of the matrix $AB$.



                The downside is that one needs to work with $12times 12$ matrices. But it is not too bad, since those matrices are essentially block-matrices.






                share|cite|improve this answer























                  up vote
                  0
                  down vote










                  up vote
                  0
                  down vote









                  I think that the problem is wrong as written, because if $a=b$, then $ab=a^2$ has minimal polynomial $x^4 - x^3 - 2 x^2 + 1$, which doesn't divide $x^6+x^4+x^3-x^2-1$ (the reminder is $7 x^3 + 6 x^2 - x - 5$).



                  I guess that it was meant that $aneq b$. Then, one may apply Galois theory to get a more straightforward way to compute the minimal polynomial of $ab$ and then to check whether or not it divides $x^6+x^4+x^3-x^2-1$ (it is a root iff the minimal polynomial divides...)



                  Here is a sketch how to do so: You prove that $mathbbQ(a,b)$ has degree $12$ and a basis $a^ib^j$ with $i=0,1,2,3$ and $j=0,1,2$. Then you write a matrix AB for multiplication by $ab$ in this basis (usually it is easier to write a matrix $A$ for $a$, a matrix $B$ for $b$, and to multiply them). The minimal polynomial polynomial will be that of the matrix $AB$.



                  The downside is that one needs to work with $12times 12$ matrices. But it is not too bad, since those matrices are essentially block-matrices.






                  share|cite|improve this answer













                  I think that the problem is wrong as written, because if $a=b$, then $ab=a^2$ has minimal polynomial $x^4 - x^3 - 2 x^2 + 1$, which doesn't divide $x^6+x^4+x^3-x^2-1$ (the reminder is $7 x^3 + 6 x^2 - x - 5$).



                  I guess that it was meant that $aneq b$. Then, one may apply Galois theory to get a more straightforward way to compute the minimal polynomial of $ab$ and then to check whether or not it divides $x^6+x^4+x^3-x^2-1$ (it is a root iff the minimal polynomial divides...)



                  Here is a sketch how to do so: You prove that $mathbbQ(a,b)$ has degree $12$ and a basis $a^ib^j$ with $i=0,1,2,3$ and $j=0,1,2$. Then you write a matrix AB for multiplication by $ab$ in this basis (usually it is easier to write a matrix $A$ for $a$, a matrix $B$ for $b$, and to multiply them). The minimal polynomial polynomial will be that of the matrix $AB$.



                  The downside is that one needs to work with $12times 12$ matrices. But it is not too bad, since those matrices are essentially block-matrices.







                  share|cite|improve this answer













                  share|cite|improve this answer



                  share|cite|improve this answer











                  answered Jul 20 at 15:30









                  Lior B-S

                  1,5661016




                  1,5661016






















                       

                      draft saved


                      draft discarded


























                       


                      draft saved


                      draft discarded














                      StackExchange.ready(
                      function ()
                      StackExchange.openid.initPostLogin('.new-post-login', 'https%3a%2f%2fmath.stackexchange.com%2fquestions%2f2857380%2fproof-involving-polynomial-roots%23new-answer', 'question_page');

                      );

                      Post as a guest













































































                      Comments

                      Popular posts from this blog

                      What is the equation of a 3D cone with generalised tilt?

                      Color the edges and diagonals of a regular polygon

                      Relationship between determinant of matrix and determinant of adjoint?